Proving "Limits of Finite Sequences Implies Limit of Sum

Click For Summary
The discussion revolves around proving that the limit of a weighted sum of a convergent sequence equals the limit of the sequence itself, under the condition that the weights approach zero. Participants suggest starting with the direction that assumes the limits of the weights are zero, leading to a conclusion that the weighted sum converges to zero. A proof by contradiction is recommended for the opposite direction, where assuming a weight does not approach zero can help demonstrate that the original limit statement fails. The conversation highlights the importance of understanding the behavior of the weights in relation to the convergence of the sequence. Overall, the proof requires careful manipulation of limits and the properties of convergent sequences.
Mr Davis 97
Messages
1,461
Reaction score
44

Homework Statement


For each ##n\in\mathbb{N}##, let the finite sequence ##\{b_{n,m}\}_{m=1}^n\subset(0,\infty)## be given. Assume, for each ##n\in\mathbb{N}##, that ##b_{n,1}+b_{n,2}+\cdots+b_{n,n}=1##.

Show that ##\lim_{n\to\infty}( b_{n,1}\cdot a_1+b_{n,2}\cdot a_2+\cdots+b_{n,n}\cdot a_n) = \lim_{n\to\infty}a_n##, for every convergent sequence ##\{a_n\}_{n=1}^\infty\subset\mathbb{R}## if and only if, for each ##m\in\mathbb{N}##, ##\lim_{n\to\infty}b_{n,m}=0##.

Homework Equations

The Attempt at a Solution


I really need at least one hint for this one. Which direction of the proof should I start with? Which one is easier?

My idea for the <---- direction is that since for each ##m\in\mathbb{N}##, ##\lim_{n\to\infty}b_{n,m}=0##, we see that ## \lim_{n\to\infty}b_{n,1}\cdot a_1+\lim_{n\to\infty}b_{n,2}\cdot a_2+\cdots+\lim_{n\to\infty}b_{n,n}\cdot a_n) = 0 + 0 + \cdots + 0##... But I'm not sure where this gets me.
 
Physics news on Phys.org
Mr Davis 97 said:

Homework Statement


For each ##n\in\mathbb{N}##, let the finite sequence ##\{b_{n,m}\}_{m=1}^n\subset(0,\infty)## be given. Assume, for each ##n\in\mathbb{N}##, that ##b_{n,1}+b_{n,2}+\cdots+b_{n,n}=1##.

Show that ##\lim_{n\to\infty}( b_{n,1}\cdot a_1+b_{n,2}\cdot a_2+\cdots+b_{n,n}\cdot a_n) = \lim_{n\to\infty}a_n##, for every convergent sequence ##\{a_n\}_{n=1}^\infty\subset\mathbb{R}## if and only if, for each ##m\in\mathbb{N}##, ##\lim_{n\to\infty}b_{n,m}=0##.

Homework Equations

The Attempt at a Solution


I really need at least one hint for this one. Which direction of the proof should I start with? Which one is easier?

My idea for the <---- direction is that since for each ##m\in\mathbb{N}##, ##\lim_{n\to\infty}b_{n,m}=0##, we see that ## \lim_{n\to\infty}b_{n,1}\cdot a_1+\lim_{n\to\infty}b_{n,2}\cdot a_2+\cdots+\lim_{n\to\infty}b_{n,n}\cdot a_n) = 0 + 0 + \cdots + 0##... But I'm not sure where this gets me.
Here is my stab at a solution for the <--- direction.

Let ##(a_n)## be an arbitrary convergent sequence that converges to ##c##. We want to show that ##b_{n,1}a_n + \cdots b_{n,n}a_n## also converges to ##c##. To this end, given ##\epsilon > 0##, there exists ##N \in \mathbb{N}## such that if ##n \ge N## we have ##|a_n - c| \le \epsilon##. ##|b_{n,1} a_1 + \cdots b_{n,n}a_n - c| = |b_{n,1} a_1 + \cdots + b_{n,n}a_n - c(b_{n,1}+ \cdots + b_{n,n})| = |b_{n,1}(a_1 - c) + \cdots b_{n,n}(a_n - c)|##... This is kind of where I get stuck. I'm not sure what to do next.
 
Mr Davis 97 said:
My idea for the <---- direction is that since for each ##m\in\mathbb{N}##, ##\lim_{n\to\infty}b_{n,m}=0##, we see that ## \lim_{n\to\infty}b_{n,1}\cdot a_1+\lim_{n\to\infty}b_{n,2}\cdot a_2+\cdots+\lim_{n\to\infty}b_{n,n}\cdot a_n) = 0 + 0 + \cdots + 0##... But I'm not sure where this gets me.
You cannot split a limit like that.

The other direction is easier if you make a proof by contradiction. Assume there is an m such that ##\lim_{n\to\infty}b_{n,m} \neq 0## and show that the original statement is no longer true. That should also give an idea how the overall structure works.
 
Question: A clock's minute hand has length 4 and its hour hand has length 3. What is the distance between the tips at the moment when it is increasing most rapidly?(Putnam Exam Question) Answer: Making assumption that both the hands moves at constant angular velocities, the answer is ## \sqrt{7} .## But don't you think this assumption is somewhat doubtful and wrong?

Similar threads

  • · Replies 7 ·
Replies
7
Views
2K
  • · Replies 7 ·
Replies
7
Views
2K
  • · Replies 13 ·
Replies
13
Views
2K
  • · Replies 8 ·
Replies
8
Views
2K
  • · Replies 7 ·
Replies
7
Views
2K
  • · Replies 8 ·
Replies
8
Views
2K
  • · Replies 2 ·
Replies
2
Views
2K
  • · Replies 1 ·
Replies
1
Views
1K
  • · Replies 15 ·
Replies
15
Views
2K
  • · Replies 34 ·
2
Replies
34
Views
3K